Ask Question
29 August, 16:20

If mean is 6 wat number could be p given se t of numbers r 4,5,8, p, 7,6

+5
Answers (1)
  1. 29 August, 18:25
    0
    Let's solve your equation step-by-step. (4+5 + 8 + p + 7 + 6) / 6 = 6 Step 1: Multiply both sides by 6. (4+5 + 8 + p + 7 + 6) / 6 = 6 (4+5 + 8 + p + 7 + 6) / (6) * (6) = (6) * (6) 4+5 + 8 + p + 7 + 6 = 36 Step 2: Simplify both sides of the equation. p+30 = 36 Step 3: Subtract 30 from both sides. p+30 - 30 = 36-30 p=6 Answer: p = 6
Know the Answer?
Not Sure About the Answer?
Find an answer to your question ✅ “If mean is 6 wat number could be p given se t of numbers r 4,5,8, p, 7,6 ...” in 📘 Mathematics if you're in doubt about the correctness of the answers or there's no answer, then try to use the smart search and find answers to the similar questions.
Search for Other Answers